Đến nội dung

tuan101293 nội dung

Có 316 mục bởi tuan101293 (Tìm giới hạn từ 10-06-2020)



Sắp theo                Sắp xếp  

#251951 phương trình nghiệm nguyên

Đã gửi bởi tuan101293 on 23-01-2011 - 07:12 trong Số học

anh đã bảo là dùng kẹp hết mà
Bài 1
1,Nếu $x\ge 1$
thì
$(x^3+1)^2<y^2=x^6+3x^3+1<(x^3+2)^2$ (vô lý)
2,Nếu $x\le -2$ thì
$(x^3+2)^2<y^2=x^6+3x^3+1< (x^3+1)^2$ (vô lý)
3,x=0 hoặc -1,thử vào suy ra x=0 là nghiệm
Bài 2
1,Nếu $x\ge 1$
thì
$x^4<y^4<(x+1)^4$ vô lý
2,Nếu $x\le -1$
thì
$x^4>y^4>(x+1)^4$ vô lý
suy ra x=0
Bài 3 thì tương tự bài 1



#251922 phương trình nghiệm nguyên

Đã gửi bởi tuan101293 on 22-01-2011 - 20:10 trong Số học

mấy bài này dùng kẹp hết chứ nhỉ???



#223138 phương trình mũ

Đã gửi bởi tuan101293 on 18-12-2009 - 19:33 trong Phương trình - hệ phương trình - bất phương trình

xin hỏi cách giải phương trình sau:
$a^x$ + $a^y$ = $a^z$ + $a^t$
mình làm là: x=z và y=t hoặc x=t và z=y có đúng không?
mình làm thử cách này với 2 đề thi đại học năm 2006 khối D đều cho đáp án đúng.
nhưng không thấy sách nào làm cách này cả?
xin mọi người góp ý.

cách làm của bạn sai hoàn toàn về bản chất đó:forall
ví dụ thế này nhé:D,y,z chọn bất kỳ.chọn $t=log_{a}(a^x+a^y-a^z)$ là thỏa mãn mà(chú ý là chọn cho cái nhân loga dương nha



#224867 Phương trình hàm đa thức

Đã gửi bởi tuan101293 on 03-01-2010 - 17:28 trong Phương trình - Hệ phương trình - Bất phương trình

Anh giải câu b,câu a thì tương tự nhé
gọi nghiệm của pt $x^2-5x+1=0$ là a,
nghiệm của $x^2-4x-1=0$ là b.
suy ra P(a)=b
đặt P(x)=(x-a)Q(x)+b thì từ điều kiện suy ra
2bQ(a)=8(a-2)Q(a) suy ra Q(a)=0.
làm tương tự.suy ra ta đặt
$P(x)=(x-a)^nL(x)+b$
trong đó L(x) ko có nghiệm=a
thì suy ra
$2bL(a)=4(2a-4)^nL(a)$ thì suy ra L(a)=0 suy ra vô lý
suy ra Q(x)=0 thì P(x)=b



#230450 Phương trình hàm liên tục

Đã gửi bởi tuan101293 on 28-02-2010 - 17:22 trong Phương trình - Hệ phương trình - Bất phương trình

x=y=0 thì f(0)=0
CM quy nạp CT sau:f(nx)=nf(x) với mọi x thuộc đoạn [0;1/n]
suy ra f(p/q)=a*p/q
đến đây chọn hai dãy con thì ta có f(x)=ax là xong



#252972 Phương trình hàm

Đã gửi bởi tuan101293 on 07-02-2011 - 20:57 trong Phương trình hàm

Hì, dạo này đãng trí, không nhớ nó là IMO Problem mặc dù làm rồi, sorry mọi người. Bài khác vậy, cũng tạm:

Tìm tất cả các hàm $f: \mathbb{R} \to \mathbb{R}$ thỏa $f(f(x)) = x^2 - 2, \forall x \in \mathbb{R}.$

bài này cũ rồi,dùng điểm bất động thôi



#252952 Phương trình hàm

Đã gửi bởi tuan101293 on 07-02-2011 - 16:17 trong Phương trình hàm

Tìm tất cả các hàm $f: \mathbb{R^+} \to \mathbb{R^+}$ thỏa mãn:

$\dfrac{(f(w))^2 + (f(x))^2}{f(y^2) + f(z^2)} = \dfrac{w^2 + x^2}{y^2 + z^2}.$


với $w, z, y, x \in \mathbb{R^+}$ và $wx = yz$.

IMO 2008 P4



#253015 Phương trình hàm

Đã gửi bởi tuan101293 on 08-02-2011 - 22:38 trong Phương trình hàm

Hì, thì em post toàn bài cũ cả. Post chơi cho vui ấy mà, có gì anh xử từ từ thôi ạ, :D...

Tìm tất cả các hàm $f: \mathbb{R} \to \mathbb{R}$ thỏa mãn $f(xf(y) + f(x)) = 2f(x) + xy$.

http://www.artofprob...f x xy#p1622377



#268284 phương trình hàm

Đã gửi bởi tuan101293 on 12-07-2011 - 21:10 trong Phương trình hàm

IMO 1999
ý tưởng là phương pháp miền giá trị
http://www.artofprob...308b8e3#p131856



#453945 phương trình $x^2+y^2+z^2=7^{2^n}$ có ít nhất $1...

Đã gửi bởi tuan101293 on 29-09-2013 - 17:42 trong Số học

ta quy nạp mệnh đề sau: ton tại bộ (x,y,z) thoả mãn điều kiện đề bài và có 2 số chẵn, 1 số lẻ.

với n = 1 ta có $$2^2+3^2+6^2 = 7^2$$

nếu có $$x^2+y^2+z^2 = 7^{2^n}$$, bộ $$(|x^2+y^2-z^2|,2xz,2yz)$$ thoả mãn (n+1).
thật vậy, lấy $$p|(x^2+y^2-z^2,2xz,2yz)$$ suy ra p lẻ
nếu $$p|z$$ suy ra $$p|x^2+y^2$$ suy ra $$p|x^2+y^2+z^2$$ suy ra p=7 vô lý
nếu z không chia hết cho p thì .... vô lý nốt
nên bộ 3 của n+1 nguyên tố cùng nhau. 
q.e.d



#222899 phuong trinh ham kho

Đã gửi bởi tuan101293 on 15-12-2009 - 13:14 trong Phương trình - Hệ phương trình - Bất phương trình

Đặt hàm f(x)=g(x)*x
suy ra $g(x)*x-g(y)*y=g(x+y)*(x-y)$
suy ra $(x-y)g(x+y)+(y-z)g(y+z)+(z-x)g(z+x)=0$
suy ra $(x-y)(g(x+y)-g(y+z))=(x-z)(g(x+z)-g(y+z))$
đặt x+y=z,y+z=b,z+x=c suy ra
$\dfrac{g(a)-g(b)}{a-b}=\dfrac{g(b)-g( c )}{b-c}=t$ với t là hăng số
suy ra g(x)=ax+b suy ra $f(x)=ax^2+bx$



#243633 Numbertheory

Đã gửi bởi tuan101293 on 12-10-2010 - 20:29 trong Số học

Tìm tất cả các số nguyên dương $n$ sao cho các số nguyên dương $a_1, a_2, ...., a_n; b_1, b_2, ..., b_n$ thỏa mãn đẳng thức:

$(a_1^2+. . .+a_n^2)(b_1^2+. . . +b_n^2)-(a_1b_1+...+a_nb_n)^2=n$

Chém thử ,chưa check kỹ lắm
ta có
$ \sum\limits_{i=1,i<j}^{n-1}(a_ib_j-a_jb_i)^2=n $
tổng trên có tối đa n phần tử khác không.
ta xét 3TH
****
TH1: $\le n-2$ số khác không
suy ra tồn tại chỉ số j sao cho $a_jbi=a_ib_j$ với mọi j suy ra theo bunhia n=0 vô lý
****
TH2: $n-1$ số khác không suy ra $n=VT\ge n-1$
suy ra $VT\ge n-2+4=n+2>n$ vô lý
****
TH3: n số khác 0
Nếu tồn tại chỉ số j như TH1 thì ta xong
Nếu không ta chia tập thành (n-1) tập {a_i} (tập a_i bao gồm tất cả các phần tử có dạng $(a_ib_j-a_jb_i)^2$)
suy ra tồn tại 1 tập có 1 phần tử=1, và 1 tập có 2 phần tử =1
xét cái tập có 1 phần tử=1 thì thấy $\dfrac{a_i}{b_i}=\dfrac{a_j}{b_j}$ (với mọi i,j trừ số hạng=1)
suy ra tất cả các tập đều có 1 phần tử=1
vô lý
****
không tồn tại n



#273372 Number theory Marathon

Đã gửi bởi tuan101293 on 21-08-2011 - 11:06 trong Số học

cho em giải bằng cách này được không ? cho em ý kiến nhé!!
vì x,y là nghiệm nguyên nên ta có: x=ky ( y thuộc Z , k thuộc R)
thay vào PT ta có:
$ 2k^4y^4+1=y^2 \Leftrightarrow y^2(1-2k^4y^2)=1 \Leftrightarrow y= \dfrac{1}{ \sqrt{1-2k^4y^2} } $
vì y thuộc Z nên $ \dfrac{1}{ \sqrt{1-2k^4y^2} } $ cũng thuộc Z
từ đó ta sẽ có $ \sqrt{1-2k^4y^2} =1 \Leftrightarrow k=0 hay y=0 $

chú giải sai chỗ này : vì y thuộc Z nên cái căn kia có dạng 1/n chứ ko fai là 1
***
bài này thì xài xuống thang thôi
China TST 1993
http://www.artofprob...dc926aa#p269097



#275278 Number theory Marathon

Đã gửi bởi tuan101293 on 05-09-2011 - 12:01 trong Số học

anh hiểu nhầm ý em rùi vì y thuộc Z nên $ \sqrt{1-4k^4y^2}$ là ước của 1 (ước của 1 gồm 1 và -1(loại)) từ đó em mới giải đây là cách giải của em nên em cũng sợ sai lắm (bài này trích ở đâu zậy mấy anh để em xem đáp án)

ô hay,
$\sqrt{1-4k^4y^2}$ có nguyên đâu, chỉ hữu tỉ thôi nên nó fai có dạng 1/n



#249851 Number

Đã gửi bởi tuan101293 on 24-12-2010 - 23:04 trong Số học

Bài này nặng về ý
CM quy nạp cái sau
Với mọi $n\in N$ tồn tại $a,b\in N$ mà
+,$(\sqrt{2}+1)^n=\sqrt{a^2}+sqrt{2b^2}$ với $a^2-2b^2=(-1)^n$
CM:
n=1,đúng
giả sử đúng tới k
ta CM đúng với k+1
chú ý $(\sqrt{2}+1)^{n+1}=(\sqrt{a^2}+sqrt{2b^2})(\sqrt{2}+1)=\sqrt{(a+2b)^2}+\sqrt{2(a+b)^2}$
và $(a+2b)^2-2(a+b)^2=-a^2+2b^2=(-1)^{n+1}$
đến đây thì ok rồi.
Merry christmas



#239120 Nice but maybe not very hard

Đã gửi bởi tuan101293 on 02-09-2010 - 15:32 trong Bất đẳng thức - Cực trị

khi đó $M\geq\dfrac{\displaystyle 1}{\displaystyle 2}[\dfrac{\displaystyle y^3}{\displaystyle x^3+y^3}+\dfrac{\displaystyle z^3}{\displaystyle y^3+z^3}+\dfrac{\displaystyle t^3}{\displaystyle z^3+t^3}+\dfrac{\displaystyle x^3}{\displaystyle t^3+x^3}]$
đến đây chỉ việc áp dụng bất đẳng thức Nesbitt cho 4 biến $x^3,y^3,z^3,t^3$ ta co ngay $M\geq1$

đây ko phải bdt nesbitt anh ơi.



#239173 Nice but maybe not very hard

Đã gửi bởi tuan101293 on 02-09-2010 - 20:13 trong Bất đẳng thức - Cực trị

Bài Toán :
Cho $4$ số thực dương $ a ; b ; c ; d $ thỏa mãn : $ abcd = 1$ ;

Chứng minh rằng ta có bất đẳng thức :

$ \dfrac{1}{(1+a)(1+a^2)} + \dfrac{1}{(1+b)(1+b^2)} + \dfrac{1}{(1+c)(1+c^2)} + \dfrac{1}{(1+d)(1+d^2)} \ge 1$
Ai đưa ra lời giải đẹp sớm nhất cho bài này ; thưởng 2$ ; Mại dzô ; mại dzô :)

Nguyễn Kim Anh

Bài này khó kinh,mình làm thử cách này(chưa check kỹ lắm,ai đó check hộ mình với)=))
đặt $f(x)=\dfrac{1}{(1+x)(1+x^2)}$ thì
**************
Bổ đề 1: nếu $ab\ge 1$

$f(a,b)\ge f(\sqrt{ab},\sqrt{ab})$ (CM=cách quy đồng trực tiếp)
và $f(a,b,c,d)\ge f(\sqrt[3]{abc},\sqrt[3]{abc},\sqrt[3]{abc})$ (CM như CM tử bdt cosi 2 số lên 3 số)
Bổ đề 2: nếu $ab\ge 1$ mà $a\ge 1$ và $b\le 1$ thì

$f(a,b)\ge f(ab,1)$ (CM=cách quy đồng)
**************
xét 3 trường hợp:
*****
TH1: 3 số nhỏ hơn 1,giả sử $b,c,d\le 1 $ suy ra $ab,ac,ad\ge 1$
ta có $f(a,b,c,d)\ge f(ab,1,c,d)\ge f(abc,d,1,1)=f(\dfrac{1}{d},d,1,1)\ge 1$
*****
TH2: 2 số nhỏ hơn 1, 2 số lớn hơn 1:
Giả sử $a\ge b\ge 1$ và $c,d\le 1$
Dễ thấy tồn tại ít nhất 1 trong 2 số ac,ad mà số này<1,giả sử $ac\ge 1$
ta có $f(a,b,c,d)\ge f(ac,b,d,1)\ge f(\sqrt{abc},\sqrt{abc},d,1)=f(x,x,\dfrac{1}{x^2},1)\ge 1 $
****
TH3: 1 số nhỏ hơn 1:
Giả sử $d\le 1$
ta có $f(a,b,c,d)\ge f(\sqrt[3]{abc},\sqrt[3]{abc},\sqrt[3]{abc},d)=f(x,x,x,\dfrac{1}{x^3})\ge 1$

ĐPCM



#239118 Nice but maybe not very hard

Đã gửi bởi tuan101293 on 02-09-2010 - 15:26 trong Bất đẳng thức - Cực trị

BDT cho n biến cũng đung đấy anh ak
$ \sum \limits_{i=1}^{n} \dfrac{1}{(a_i+1)(a_{i}^2+1)} \ge \dfrac{n}{4}$
Ta sẽ cm BDT sau: $\dfrac{1}{(1+a)(1+a^2)} \ge \dfrac{1}{4}-\dfrac{3}{8}ln a$
Thât vậy $f(x)=\dfrac{1}{(1+x)(1+x^2)} -\dfrac{1}{4}+\dfrac{3}{8}ln x$ :)
thì $f'(x)=\dfrac{(x-1)(3x^6+9x^5+18x^4+30x^3+39x^2+21x+8)}{8(1+x)^2(1+x^2)^2}$
i)$x \ge 1 \to f(x) \ge f(1)=0$
ii)$0<x<1 \to f(x) \ge f(1)=0$
Tóm lại :) đúng
Cộng từng vế có QED

PS: 2$ ???? :) :) USD hôm nay 19,5

Đạo hàm sai rồi em zai.
$f'(x)=\dfrac{(x-1)(3x^5+9x^4+18x^3+6x^2-x-3)}{8(1+x+x^2+x^3)^2x}$



#238986 Nice

Đã gửi bởi tuan101293 on 01-09-2010 - 19:59 trong Bất đẳng thức và cực trị

$a,b,c>0$
$ab+bc+ca=3$
Chung minh: $ \sum \dfrac{a}{2 a^{2}+bc } \geq abc$

đẹp và dễ thật :)
ta có bdt tương đương (chia abc cho 2 vế)
$\sum \dfrac{1}{2a^2bc+b^2c^2}\ge 1$ đặt ab=z,bc=x,ca=y suy ra x+y+z=3
và ta phải CM
$\sum \dfrac{1}{x^2+2yz}\ge \dfrac{9}{x^2+y^2+z^2+2xy+2yz+2zx}=1$
ĐPCM



#243674 nice

Đã gửi bởi tuan101293 on 13-10-2010 - 13:11 trong Số học

Tìm tất cả các số nguyên dương $a,b,c$ thỏa mãn:

$\dfrac{a^2+b^2+c^2}{3(ab+bc+ca)}$ là số nguyên.

suy ra
$(a+b+c)^2=(3n+2)(ab+bc+ca)$
tồn tại số nguyên tố p=3k+2 sao cho
$p^{2t-1}||(3n+2)$
chú ý vế trái là SCP
nên $p^{2t}|(a+b+c)^2$ nên $p|a+b+c$
suy ra $p|ab+bc+ca$
nên $p|a^2+ab+b^2|(2a+b)^2+3b^2$
suy ra -3 là SCP mod p

$p \equiv 2(mod 3)$
suy ra vô lý
không tồn tại n



#245266 nice

Đã gửi bởi tuan101293 on 25-10-2010 - 18:54 trong Số học

+,Vì nếu tất cả các ước nguyên tố của (3n+2) có dạng (3k+1) thì xét mod 3 ta suy ra vô lý.Nên tồn tại ước có dạng (3k+2) và số mũ của ước này là lẻ,nếu ko lại quy về TH trước.
+,$c\equiv -(a+b) (mod p)$ thì $c(a+b)+ab\equiv -(a+b)^2+ab (mod p)$ nên $p|a^2+ab+b^2$



#258545 Nhờ giúp một BĐT

Đã gửi bởi tuan101293 on 20-04-2011 - 07:19 trong Bất đẳng thức - Cực trị

1. Cho $p_1 = 2,p_2 = 3,p_3 = 5,...,p_n $ là n số nguyên tố đầu tiên, với n :( 3. CMR:
$\dfrac{1}{{p_1^2 }} + \dfrac{1}{{p_2^2 }} + \dfrac{1}{{p_3^2 }} + ... + \dfrac{1}{{p_n^2 }} + \dfrac{1}{{p_1 p_2 ...p_n }} < \dfrac{1}{2}$

*********
LG:
có $p_n\ge 2n-1$ với mọi n (do trong 2 số liên tiếp ta chỉ có nhiều nhất 1 số nguyên tố (1 chẵn 1 lẻ ))
và do $p_1p_2...p_n>2*3*5*7$ với n>4
nên ta có

$VT=\dfrac{1}{2^2}+\dfrac{1}{3^2}+\dfrac{1}{5^2}+\dfrac{1}{7^2}+\dfrac{1}{p_5^2}+....+\dfrac{1}{p_n^2}+\dfrac{1}{p_1....p_n}<\dfrac{1}{2^2}+\dfrac{1}{3^2}+\dfrac{1}{5^2}+\dfrac{1}{7^2}+(\dfrac{1}{9^2}+....+\dfrac{1}{(2n-1)^2})+\dfrac{1}{2*3*5*7}<\dfrac{1}{2^2}+\dfrac{1}{3^2}+\dfrac{1}{5^2}+\dfrac{1}{7^2}+(\dfrac{1}{7*9}+....+\dfrac{1}{(2n-3)(2n-1)})+\dfrac{1}{2*3*5*7}=\dfrac{1}{2^2}+\dfrac{1}{3^2}+\dfrac{1}{5^2}+\dfrac{1}{7^2}+\dfrac{1}{2}(\dfrac{1}{7}-\dfrac{1}{9}+....+\dfrac{1}{2n-3}-\dfrac{1}{(2n-1)})+\dfrac{1}{2*3*5*7}<\dfrac{1}{2^2}+\dfrac{1}{3^2}+\dfrac{1}{5^2}+\dfrac{1}{7^2}+\dfrac{1}{2*7}+\dfrac{1}{2*3*5*7}<\dfrac{1}{2}$
ĐPCM



#222567 Nhập môn học giải tích!

Đã gửi bởi tuan101293 on 08-12-2009 - 17:48 trong Giải tích

Vì mới nhập môn nên em thấy bài tập hơi khó, mong các anh giúp em 2 bài.

Bài 1:

a) ĐÃ TỰ LÀM ĐC

b) lim $( x^{3} - x^{2} + x + 6 log_{2}x) 2^{x}$
x->+0

b,Nếu đề câu b thế kia thì lim là $-\infty $
anh check lại đề đi



#252124 nguyên hàm đây zô cho zui

Đã gửi bởi tuan101293 on 25-01-2011 - 21:59 trong Tích phân - Nguyên hàm

Không ai ủng hộ topic của tui sao buôn quá trời. Hay là chưa đủ khó để làm vậy thì tiếp nè .....

1.Nguyên hàm $\int {\left( {\dfrac{1}{{\ln ^2 }} + \dfrac{1}{{\ln x}}} \right)} dx$

2.$\int {\dfrac{1}{{\ln ^2 }}dx} $ và $\int {\dfrac{1}{{\ln x}}dx} $

Mình chỉ biết cái nguyên hàm này thôi

$ \int (\dfrac{1}{ln(x)}-\dfrac{1}{ln(x)^2})dx =\dfrac{x}{ln(x)}$
mấy cái kia có lẽ là ko tính được đâu



#454632 nghịch lý monty hall

Đã gửi bởi tuan101293 on 02-10-2013 - 14:13 trong IQ và Toán thông minh

cái này là conditional probability rõ ràng, sự kiện sau liên quan đến sự kiện trước, bạn rai_2601 nói không đúng nhé.

Cm thì có 1 bạn ở trên nói nghe 33+33=66 cũng đúng  nhưng để lập luận toán học thì người ta thường dùng Bayes theorem. có 1 dòng thôi à.

trước mình học cái này cũng hơi bất ngờ. Nhuwng hài ở chỗ là ai cũng nghĩ là 50% thành ra chơi ô cửa bí mật chả ai đổi =)).